HELP ASAP PLEASE 20 POINTS!!!!!

HELP ASAP PLEASE 20 POINTS!!!!!

Answers

Answer 1

Answer:

5.6 mi

Step-by-step explanation:

1 km is around 0.6 (estimate) miles so just multiply both sides by 9 and 9km = around 5.4 miles and 5.6 is the closest so it is most likely that


Related Questions

When a retired police officer passes away, he leaves $300,000 to be divided among his three children and six grandchildren. The will specifies that each child is to get twice as much as each grandchild. How much does each get?

Answers

Answer:

so first off if its six grandchildren and 3 children then that means 9 people but if the children get double the amount distributed then lets treat it like 12 people so 25,000 each meaning each child of the police officer gets 50 thousand and each grandchild gets 25 thousand

Step-by-step explanation:

hope this helps !

Each child will get 50,000 and each grandchild will get 25,000

At which values in the interval [0, 2π) will the functions f (x) = cos 2x + 1 and g(x) = sin x + 2 intersect?

x equals 0 comma pi over 6 comma 5 times pi over 6 comma pi
x equals 0 comma pi comma 7 times pi over 6 comma 11 times pi over 6
x equals pi over 2 comma 7 times pi over 6 comma 3 times pi over 2 comma 11 times pi over 6
x equals pi over 6 comma pi over 2 comma 5 times pi over 6 comma 3 times pi over 2

Answers

The functions f(x) = cos 2x + 1 and g(x) = sin x + 2 intersect at x = pi/6, pi/2, 5pi/6, and 3pi/2 in the interval [0, 2π).

To find the intersection points, we set f(x) equal to g(x) and solve for x.

cos 2x + 1 = sin x + 2

Rearranging, we get:

cos 2x = sin x + 1

Using the identity cos 2x = 1 - 2sin²x, we can rewrite this as:

1 - 2sin²x = sin x + 1

Simplifying and factoring, we get:

2sin²x + sin x = 0

sin x(2sin x + 1) = 0

Therefore, sin x = 0 or sin x = -1/2. Solving for x in the interval [0, 2π), we get x = pi/6, pi/2, 5pi/6, and 3pi/2. These are the values at which f(x) and g(x) intersect.

learn more about intersection points here:

https://brainly.com/question/29188411

#SPJ1

The administrator of a county's museum is considering increasing the price of an annual pass by 1% from $25 to $25.25. At the current price, the museum sells 2500 annual passes and the point of elasticity is 0.73. What effect would the increase in price have on the museum's annual pass revenue?

Answers

Answer: the increase in price from $25 to $25.25 would result in a small increase in annual pass revenue of $178.50 or about 0.3%.

Step-by-step explanation: we can calculate the modern yearly pass income by increasing the modern amount requested by the unused cost of $25.25:

Unused yearly pass income = Modern cost x Modern amount requested

Unused yearly pass income = $25.25 x 2482 = $62,678.50

We as of now know that the ancient yearly pass income is $62,500 (since 2500 passes were sold at $25 each), so ready to calculate the alter in yearly pass income:

Alter in yearly pass income = Unused yearly pass income - Ancient yearly pass income

Alter in yearly pass income = $62,678.50 - $62,500 = $178.50

If it takes 15 minutes for water in a pot to boil, the expression 20x+ 15 can be used to find the total time needed to cook x batches of boiled cornbread. Which expression can also be used to determine the total time needed to cook x batches of boiled cornbread? PLS ANSWER ASAPP I REALLY NEED TO ANSWER THIS QUESTION

A 18x + 4x + 6 + 7
B 10x + 10x + 20 - 5
C 6(5x + 5) - 10x + 15
D 3(x + x + x + 3)+ 2x + 15

Answers

the correct expression which can also be used to determine the total time needed to cook x batches of boiled cornbread is,

B) 10x + 10x + 20 - 5.

Given that;

The expression 20x + 15 represents the total time needed to cook x batches of boiled cornbread.

We can simplify this expression by factoring out the common factor of 5: = 20x + 15

= 5(4x + 3)

Therefore, the expression that can also be used to determine the total time needed to cook x batches of boiled cornbread must have a factor of 5 in it.

Option A:

18x + 4x + 6 + 7 does not have a factor of 5,

so it cannot be the correct answer.

Option B:

10x + 10x + 20 - 5

= 20x + 15,

which is the same as the original expression.

Therefore, Option B is equivalent to the original expression and can also be used to determine the total time needed to cook x batches of boiled cornbread.

Option C:

6(5x + 5) - 10x + 15 = 30x + 15 - 10x + 15 = 20x + 30,

which does not have a factor of 5.

Therefore, Option C is not correct.

Option D:

3(x + x + x + 3) + 2x + 15 = 9x + 6 + 2x + 15 = 11x + 21,

which does not have a factor of 5.

Therefore, Option D is not correct.

Therefore, the correct answer is,

B) 10x + 10x + 20 - 5.

Learn more about the mathematical expression visit:

brainly.com/question/1859113

#SPJ1

please help with this question! ​

Answers

Answer:

The true statements:

6 is an element of D

12 is an element of D

3.4 MIXED FACTORING

1. Utilize all of the strategies for factoring in order to factor the following polynomials.
Reminder: Combine like-terms prior to factoring.

(2x^2 + 23x) - (3x - 18)

Answers

Answer:

2 (x + 1)(x + 9)

Step-by-step explanation:

The given expression is
[tex]\left(2x^2\:+\:23x\right)\:-\:\left(3x\:-\:18\right)[/tex]

and we are asked to factor it

Step 1
Remove parentheses
[tex]\left(2x^2\:+\:23x\right) = 2x^2 + 23x\\\\-\:\left(3x\:-\:18\right) = - (3x) - (-18) = -3x + 18\\\\[/tex]

Step 2
Add the individual terms to correspond to the original expression:
[tex]\left(2x^2\:+\:23x\right)\:-\:\left(3x\:-\:18\right) \\\\= 2x^2 + 23x -3x + 18\\\\= 2x^2 + 20x + 18[/tex]

Step 3
Factor out common term 2

[tex]2x^2+20x+18 \\\\= 2\left(x^2+10x+9\right)[/tex]

Step 4
Factor [tex]x^2+10x+9[/tex]

To do this find two numbers such that their sum = 10 and product = 9
We can easily see that these two numbers are 1 and 9 because 1 + 9 = 10 and 1 x 9 = 9

Therefore, splitting the expression into groups we get
[tex]x^2 + 10x + 9 = x^2 + 1x + 9x + 9\\\\[/tex]

Step 5

Factor:

[tex]x^2 + 1x = x(x+ 1)\\9x + 9 = 9(x + 1)\\\\[/tex]

Therefore
[tex]x^2 + 1x + 9x + 9 = x(x + 1) + 9(x + 1)\\\\[/tex]

Step 6

Factor common term (x + 1) from the expression
[tex]x(x + 1) + 9(x + 1) = (x + 1)(x + 9)[/tex]

Step 7
Putting it all together
Remember we factored out the 2 in step 3 so we got to put it back into the factored expression giving the final factored expression as

[tex]2 (x + 1)(x + 9)[/tex]

100 POINTS

Question:

Answers

read the paragraph and then you’d have all the information that you need

Answer:

First braclets coast 3$ and necklaces coast 6$ so now in the box you can put 3in the first one then 6 in the next and keep going its like multiples of 3 and 6 :) Hope this helps.

Step-by-step explanation:

If the number 48700 is written in scientific notation, it will be in the form 4.87×10n. What is the value of n?

Answers

If the number 48700 is written in scientific notation, it will be in form 4.87×10ⁿ, and the value of n is 4.

What is scientific notation?

Scientific notation describes the shorthand way of writing very large or very small numbers in standard forms.

Writing in scientific notation, a number between 1 and 10 is multiplied by a power of 10.

48700 = 4.87×10ⁿ = 4.87×10⁴

Thus, the n represents the exponent of power 10 to which the number is reduced.

Learn more about scientific notations at https://brainly.com/question/27862246.

#SPJ1

Write an equation that represents the line.

Answers

Answer:

y = (3/4)x + 2

Step-by-step explanation:

A line (or linear function) can be represented by the equation y = mx + b where "m" is the slope and "b" is the y-intercept.

The y-intercept is when x = 0 or when the line crosses the y-axis. In the graph, notice that the line crosses at (0, 2). Thus, the y-intercept is at y = 2 and "b" must be "2".

Next, we have to find the slope, "m". The slope is defined as a change in "y" divided by a change in "x". Lets use the two points which are marked on the graph already, (4, 5) and (0, 2). We first subtract the y-values, and then divide that by the difference between corresponding x-values:

(5 - 2)/(4 - 0) = 3/4                           The slope is 3/4, so m = 3/4

Now, plug "m" and "b" into the equation:

y = (3/4)x + 2

Answer:

y=(3/4)x+2

Step-by-step explanation:

So first you need to find the slope (3/4). To find that you can use the rise over run method. In this case they have already chosen two points on the line, but you could use any exact points. Count how many up from 2 to 5 (in green), which is 3 then how many across (in red) which is 4. Then put the rise (3) over the run (4) so 3/4.

Then b (2) is equal to y in the y intercept which is (0,2)

A study of the consultants in a particular industry has determined that the standard deviation of the hourly fee of the consultants is $23. A random sample of
100 consultants in the industry has a mean hourly fee of $113. Find a 99% confidence interval for the true mean hourly fee of all consultants in the industry.
Then give its lower limit and upper limit.
Carry your intermediate computations to at least three decimal places. Round your answers to one decimal place. (If necessary, consult a list of formulas.)
Lower limit:
Upper limit:
X
5

Answers

If a study of the consultants in a particular industry has determined that the standard deviation of the hourly fee of the consultants is $23.  the 99% confidence interval for the true mean hourly fee of all consultants in the industry is (107.1, 118.9).

How to find the confidence interval?

The formula for a confidence interval for the population mean is:

CI = X- bar ± z* (σ/√n)

where:

X- bar  = sample mean

z* = z-score for the desired level of confidence (99% in this case)

σ = population standard deviation

n = sample size

Substituting the given values, we get:

CI = 113 ± 2.576 * (23/√100)

Calculating the standard error:

SE = σ/√n = 23/√100 = 2.3

Now we can substitute this value into the formula:

CI = 113 ± 2.576 * 2.3

CI = 113 ± 5.92

So,

Lower limit: 107.1

Upper limit: 118.9

Therefore, the 99% confidence interval for the true mean hourly fee of all consultants in the industry is (107.1, 118.9).

Learn more about confidence interval here:https://brainly.com/question/15712887

#SPJ1

1. A rectangular block of wood has dimensions 24 cm by 9 cm by 7 cm. It is cut up into children's bricks. Each brick is a cube of side 3 cm.
(a) Find the largest number of bricks that can be cut from the block.
(b) Find the volume of wood that is left. ​

Answers

The dimensions of the rectangular block indicates;

(a) The largest number of bricks that can be cut from the block is 48 bricks

(b) The volume of the wood left is 216 cm³

What is the volume of a rectangular block?

The volume of the rectangular block is the product of the length, L, width, W, and height, H.

1. The dimensions of the rectangular block indicates that the block volume is; V = 24 cm × 9 cm × 7 cm = 1,512 cm³

The largest number of bricks that have side lengths of 3 cm that can be cut from the block is therefore;

Number of cubes; (24)/3 × (9)/3 × (7)/3 ⇒ 8 × 3 × 2 = 48

48 cube bricks can be cut from the block

(b) The volume of wood left = 1512 - 48 × 3 × 3 × 3 = 216

The volume of wood left is 216 cm³

Learn more on the volume of regular solids here: https://brainly.com/question/31010344

#SPJ1

Asynchronous activity

Answers

In the triangles given the analysis for same is given below

What is the analysis of the two triangles given above?

1) the corresponding vertices are

R ~ C and

E ~ 0

2) The corresponding angle are

∠E ~∠O  and

∠ R ~ ∠C

3)

The corresponding sides are;

ER ~ CO;
MO ~ EM
CM ~ RM

4)  the congruent angles are:

∠REM ≅ ∠MOC
∠ERM ≅ ∠MCO
∠CMO ≅ EMR

5) the congruent sides are:

ER ≅ CO;
MO ≅ EM
CM ≅ RM

6) the congruent triangles are;

ΔMER ≅ ΔMOC

Learn more about triangles:
https://brainly.com/question/2773823
#SPJ1

A certain test preparation course is designed to help students improve their scores on the LSAT exam. A mock exam is given at the beginning and end of the course to determine the effectiveness of the course. The following measurements are the net change in 5 students' scores on the exam after completing the course:

−5,19,4,0,1
Using these data, construct a 80% confidence interval for the average net change in a student's score after completing the course. Assume the population is approximately normal.


Step 3 of 4 : Find the critical value that should be used in constructing the confidence interval. Round your answer to three decimal places.

Answers

The critical value for an 80% confidence interval is 1.28 (rounded to three decimal places).

What is the statistics?

Statistics is a branch of mathematics that involves the collection, analysis, interpretation, presentation, and organization of data.

To find the critical value for an 80% confidence interval, we need to determine the z-score corresponding to the level of confidence. We can use a standard normal distribution table or a calculator to find this value.

Using a standard normal distribution table, we can find the z-score that corresponds to an 80% confidence level by finding the area between the mean and the upper tail of 10%, which is (100% - 80%)/2 = 10%.

The area between the mean and 10% is 0.5 - 0.10 = 0.40. Using the standard normal distribution table, the z-score that corresponds to an area of 0.40 is approximately 1.28.

Hence, the critical value for an 80% confidence interval is 1.28 (rounded to three decimal places).

To learn more about z-score, Visit

https://brainly.com/question/15222372

#SPJ1

50 Points! Algebra question. Photo attached. Please show as much work as possible. Thank you!

Answers

Answer:

the answer for this question is x=8

The Morning Gazette offers employees 1.65% of the average of their last 3 years of annual compensation for each year of service. Rita began working for the Morning Gazette in 1994. She retired in 2016. In 2014, she made $76,000 per year. Thereafter, she received a 3% salary increase each year until she retired.
a) How much did she earn for each year from 2014 through 2016?
b) What is the average of her last five years of working?
c) How much was his annual retirement benefit?

Answers

She earn fοr each year frοm 2014 thrοugh 2016 is 80628.40. The average οf her last five years οf wοrking $78,302.80. His annual retirement benefit was $28,423.92.

a) Salary οf 2014 : $76000

Salary in 2014 is the salary in 2015 increased by 3% οf the salary

= $76,000+3%

Salary οf 2015 = $78,280

Salary in 2015 is the salary in 2016 increased by 3% οf the salary

=$78,280+3%

Salary οf 2016 = $80,628.40

Hence, she earn fοr each year frοm 2014 thrοugh 2016 is 80628.40

b) The average οf the last three years is the sum οf the salaries divided by the number οf salaries.

= $76,000+$78,280+$80,628.40 / 3

= $78,302.80

Hence, the average οf her last five years οf wοrking $78,302.80

c ) The annual retirement benefit is the prοduct οf the rate and the average and the number οf years οf service.

= 1.65%×$78,302.80×22

=$28,423.92

Hence,  his annual retirement benefit was $28,423.92.

To know more about salary check below link:

https://brainly.com/question/28920245

#SPJ1

Boyd's board shop grt a shipment of 92 new skateboards wheels. Boyd assembles the skateboards and puts 4 of the new wheels on each board. You can use a function to describe the number of new wheels Boyd has left after assembling x skateboards

Answers

The function to describe the number of new wheels Boyd has left is f(x) = 92 - 4x


Calculating the function to describe the number of new wheels Boyd has left

From the question, we have the following parameters that can be used in our computation:

Shipment of 92 new skateboards wheels. 4 of the new wheels on each board.

After assembling x skateboards, we have

f(x) = Shipmnet - Rate * x

Substitute the known values in the above equation, so, we have the following representation

f(x) = 92 - 4 * x

Evaluate

f(x) = 92 - 4x

Hence, the function is f(x) = 92 - 4x

Read more abiut linear relation at

https://brainly.com/question/30318449

#SPJ1

A political pollster wants to know what proportion of voters are planning to
vote for the incumbent candidate in an upcoming election. A poll of 150
randomly selected voters is taken from the more than 2,000 voters in the
population, and 78 of those selected plan to vote for the incumbent
candidate.
Based on this sample, which of the following is a 90% confidence interval
for the proportion of all voters who plan to vote for the incumbent
candidate?
Choose 1 answer:
780.067
78±0.080
0.520.067

Answers

The confidence interval is 0.52±0.067.(option c)

What is confidence interval?

In statistics,  confidence interval means that the probability in which a population parameter will fall between a set of values for a certain proportion of times.

A political pollster wants to know what proportion of voters are planning to

vote for the incumbent candidate in an upcoming election. A poll of 150

randomly selected voters is taken from the more than 2,000 voters in the

population, and 78 of those selected plan to vote for the incumbent

candidate.

The formula for confidence interval(CI) is given by:

                      CI=  p ± z √{p(1-p)/n} ----------------(1)

Here from the given data p= 78/150= 0.52

                                          n= 150

For 90% confidence interval the value of z= 1.645.

Putting all the values in equation (1) we get,

                         CI= 0.52± 1.645√{(0.52×0.48)/150}

                             = 0.52±1.645×0.04

                             = 0.52±0.067

Hence, the confidence interval is 0.52±0.067.

To know more about confidence interval

https://brainly.com/question/20309162      from the link.

#SPJ9

Which of the following numbers is shown plotted on the number line?
0
0 2.75 0 2 0-3 0-2 0-15

Answers

Answer:

-3 3/8

Step-by-step explanation:

The only number we're given on the number line is 0, but we can assume that each tick on the number line represents 1 units.  We see that between the ticks that are 4 and 3 spaces away from 0.Since the point is to the left of 0, we know the number must be negative.The only answer that would make sense is -3 3/8, since the tick 4 places away from 0 is -4 and the tick 3 units away from 0 is -3 and -3 3/8 lies between -3 and -4

 Cooper can purchase a item for $45.00, Which store has the best buy?

A. Store 1: 25% off coupon
B. Store 2: $15 dollars off
C. Store 3: 10% off

I’ll give brainly please help me

Answers

Store 2 has the best price

What did Maleuvre write about Gauguin's views on Polynesian women?

Answers

Answer:

Step-by-step explanation:

Observation of the real life and way of life of the peoples of Oceania are intertwined in them with local myths.

Jackson's Ice Cream Shop must have a mean of 110 visitors per day in order to make a profit. The table below shows the number of visitors during 1 week.

Day of the week | No of Visitors

Sunday.................... 63

Monday.................. 77

Tuesday.................. 121

Wednesday............... 96

Thursday .......................137

Friday................................ 154


Based on the table, how many visitors will need to go to Jackson's Ice Cream shop on Saturday in order for the company to make a profit for the week?


A-108

B-110

C-122

D-222


Please add work/ explain!!

Answers

To find out how many visitors Jackson's Ice Cream Shop needs on Saturday to make a profit for the week, we need to first calculate the total number of visitors for the six days already given in the table. Then we can find out the number of visitors needed on Saturday to reach the required mean of 110 visitors per day for the week.

Total number of visitors for the six days = 63 + 77 + 121 + 96 + 137 + 154 = 648

To make a profit for the week, the total number of visitors for the entire week needs to be:

110 visitors/day × 7 days/week = 770 visitors/week

We already have 648 visitors for the six days, so the number of visitors needed on Saturday to make a profit can be found by subtracting the total number of visitors already counted from the total number of visitors needed for the week:

770 - 648 = 122

Therefore, the answer is (C) 122 visitors on Saturday are needed to make a profit for the week.

The value of x in the following equation 4x-5=7 is .........​

Answers

Answer:

Step-by-step explanation:

4x - 5 = 7

4x = 7 + 5

4x = 12

x = 12/4

x = 3

Please solve this for me!

Answers

Answer: 1; 520/9 number 2: 9/2 number 3: d number 4:d number 5: 151.7

Step-by-step explanation:

please answer the question in pic

Answers

Note that given the factors above, Orr should replace the above plant. This has to do with depreciation and book value.

Why is this so  ?

We need to first compute the current total cost of keeping the old plant.

Book value = $4,500,000 - $1,000,000/year x 4 years

= $500,000

Thi smean that the current total cost of keeping the old plan tfor the reamining 10 year is

Total cost of keeping old plant = $1,000,000/year x 10 years - $500,000 + $150,000

= $9 650 000

Now lets calculate the total cost of purchasing the new plant for 10 years.

$600,000 - $150,000

= $450,000 per year.

The salvage value after 10 years is $0.

Total cost of new plant = $3,500,000 + $450,000/year x 10 years = $8,000,000

Since the cost of keeping is higher than the new one, then Orr should get a new plant.

$9 650 000 > $8,000,000

Learn more about depreciation:
https://brainly.com/question/30531944
#SPJ1

How large a sample is needed in order to be 90% confident that the sample proportion will not differ from the true proportion by more than 3%? a previous study indicates that the proportion is 21%

Answers

Using the data given such as the margin of error, sample size and z-score, the maximum sample size required is 499

How large a sample is needed in order to be 90% confident that the sample proportion will not differ from the true proportion by more than 3%?

To solve this problem, we need to know the minimum sample size required. The formula for this is given as;

n = (z² * p(1 - p))/ E²

n = sample sizeE = Maximum margin of errorp = estimated proportion from the given previous studyz = z-score

Substituting the values into the formula;

n = (1.645² * 0.21(1 - 0.21))/ 0.03²

n = 498.8

The sample size should be at least approximately 499

Learn more on sample size here;

https://brainly.com/question/28583871

#SPJ1

The function h(t) = 4 + 64t – 16t2 models the height h, in feet, of a ball thrown in the air, after t seconds.

Part A

What is the vertex of the graph of the function, (t, h(t))?

(
,
)

Part B

What does the t-coordinate of the vertex represent?
A. the ball's maximum height
B. the time it takes for the ball to reach its maximum height
C. the time it takes for the ball to hit the ground
D. the height the ball was thrown from
Part C

What does the h(t)-coordinate of the vertex represent?
A. the ball's maximum height
B. the time it takes for the ball to reach its maximum height
C. the time it takes for the ball to hit the ground
D. the height the ball was thrown from

Answers

Part A) The vertex of the graph of the function, (t, h(t)) is (2, 68).

Part B) The t-coordinate of the vertex represent is the time it takes for the ball to reach its maximum height (option b)

Part C) The h(t)-coordinate of the vertex represent is the ball's maximum height (option a).

Part A asks for the vertex of the graph of the function, which is the point where the function reaches its maximum or minimum value. To find the vertex of a quadratic function like

=> h(t) = 4 + 64t – 16t²,

we can use the formula

=>  t = -b/2a,

where a, b, and c are the coefficients of the quadratic equation ax² + bx + c and t is the input variable (in this case, the time).

The t-coordinate of the vertex is simply the value we get when we plug this formula into our equation.

So, for

=> h(t) = 4 + 64t – 16t²,

we have a = -16, b = 64, and c = 4.

Plugging these values into the formula

=> t = -b/2a,

we get

=> t = -64/(2*(-16)) = 2.

The t-coordinate of the vertex is therefore 2.

To find the h(t)-coordinate of the vertex, we can simply plug t = 2 into the function h(t) = 4 + 64t – 16t² and evaluate it.

This gives us

=> h(2) = 4 + 64(2) – 16(2²) = 68.

Therefore, the vertex of the graph of h(t) is (2, 68).

Part B asks what the t-coordinate of the vertex represents. We know that the t-coordinate is the time at which the ball reaches its maximum height.

Therefore, the correct answer is B: the time it takes for the ball to reach its maximum height.

Part C asks what the h(t)-coordinate of the vertex represents. We just found that the h(t)-coordinate of the vertex is the maximum height of the ball.

Therefore, the correct answer is A: the ball's maximum height.

To know more about function here

https://brainly.com/question/28193995

#SPJ1

express 7 min 30 sec. as a percentage of 1 hour​

Answers

Answer:

12.5 is the answer

Step-by-step explanation:

7 × 60 ( 7 being the minutes, 60 being the hour )

= 420

+ 30

= 450

450 ÷ 3600 × 100

45000 ÷ 3600

450 ÷ 30

= 12.5

12.5 being the answer

Answer: 12.5%

Hope this helps! Good luck!

PLS HURRY I AM GIVING BRAINLIEST!!!
the question is in the photo!!

Answers

The sum of both sides is 2x² + 4. The length of the third side is 5x³ - 4x² + 3x - 12

What is a triangle?

A triangle is a polygon that has three sides and three angles. Types of triangles are isosceles, right angled, scalene, equilateral.

Given that the perimeter of the triangle is 5x³ - 2x² + 3x - 8. The length of side one is 3x² - 4x - 1 and that of side two is -x² + 4x + 5

The sum of side one and side two is:

Sum = side 1 + side 2 = (3x² - 4x - 1) + (-x² + 4x + 5) = 2x² + 4

The sum of both sides is 2x² + 4

The third side is:

Third side = Perimeter - Sum of two sides = (5x³ - 2x² + 3x - 8) - (2x² + 4) = 5x³ - 4x² + 3x - 12

The length of the third side is 5x³ - 4x² + 3x - 12

Find out more on triangle at: https://brainly.com/question/28470545

#SPJ1

How much money should Be deposited today in an account that earns 10.5 percent compounded monthly so that it will accumulate to $22,000 in four years

Answers

Answer:

$12,276.24

Step-by-step explanation:

22000 / (1 + 0.105/12)^4*12

12276.24

We can use the formula for compound interest to solve this problem:

A = P(1 + r/n)^(nt)

where A is the future value, P is the present value, r is the annual interest rate, n is the number of times the interest is compounded per year, and t is the number of years.

In this case, we want to find the present value P that will accumulate to $22,000 in four years at an annual interest rate of 10.5% compounded monthly (i.e., n = 12). Therefore, we have:

A = $22,000
r = 0.105/12 (monthly interest rate)
n = 12
t = 4

Substituting these values into the formula and solving for P, we get:

P = A/(1 + r/n)^(nt) = $22,000/(1 + 0.105/12)^(12*4) ≈ $14,005.70

Therefore, the amount that should be deposited today in the account is approximately $14,005.70.

Given the two concentric circles with center X below, find the area of the shaded region. Round your answer to the nearest tenth if necessary.

Answers

The area of the shaded region is given as follows:

298.3 units².

How to calculate the area of a circle?

The area of a circle of radius r is given by the multiplication of π and the radius squared, as follows:

A = πr²

The larger circle has a radius of 7 + 5 = 12 units, hence it's area is given as follows:

A = 3.14 x 12²

A = 452.16 units².

The smaller circle has a radius of 7 units, hence it's area is given as follows:

A = 3.14 x 7²

A = 153.86 units².

Hence the area of the shaded region is given as follows:

452.16 - 153.86 = 298.3 units².

More can be learned about the area of a circle at https://brainly.com/question/15673093

#SPJ1

Other Questions
Ou are constructing a portfolio of two assets, asset a and asset b. The expected returns of the assets are 10 percent and 15 percent, respectively. The standard deviations of the assets are 22 percent and 30 percent, respectively. The correlation between the two assets is. 27 and the risk-free rate is 4 percent. What is the optimal sharpe ratio in a portfolio of the two assets? what is the smallest expected loss for this portfolio over the coming year with a probability of 5 percent? (a negative value should be indicated by a minus sign. Do not round intermediate calculations. Round your sharpe ratio answer to 4 decimal places and the z-score value to 3 decimal places when calculating your answer. Enter your smallest expected loss as a percent rounded to 2 decimal places. ) What differences/adaptations give some organisms a better chance of surviving in their environment Essay about why is soccer your favorite sport Please help me i really dont get this. ill give 20 points please help me. ILL GIVE BRAINLIEST What was the effect of the addition of FeCl3 to the sample solution in the dichromate titration? Explain Mg(s) + HCl(aq) --->a. Predict the productsb. Balance the equationc. Explain what is going on in the reaction using a sentence equation. d. How many moles of HCl are consumed by the reaction of 1. 54 moles of magnesium?e. How many moles of Hare produced from gas when 2. 56 x 10(-7) grams of HCl is added to the reaction?f. How many grams of magnesium are needed to react with 0. 03 moles of hydrochloric acid?g. How many grams of hydrogen gas gets produced from 7. 92 grams of magnesium?(WOULD MEAN ALOT IF SOMEONE CAN PLEASE ASSIST ME WITH THIS) The amount of time spent practicing shooting free throws and the percentage made in a game Very confusing help Im not sure how break this down and whats the step what does new thousand mean? Scenario E: Aspen Integrated Marketing used to have a strict hierarchical structure, with information given only to those who required it. The new chief executive officer, however, set up a flat organizational structure that eliminates barriers to information flow. Information that was previously available to managers alone is now given to employees as well. He also assigned mentors to new employees to help them in their jobs and enable them to perform better. The process by which mentors help new employees with their jobs and enable them to perform better is an example of The USA PATRIOT Act, passed after the September 11, 2001 attacks, was designed to help law enforcement intercept terrorists. Somelawmakers opposed this law because they were concerned that it -took too much power away from the executive branchwas another example of abuses like the War Powers Actcould result in violations of citizens' constitutional rightsdid not include clear enough guidelines for law enforcement Use the picture to help. Will brainlist.What occupations can you not expect to see in the negative projected growth area? WhyWhat occupations did you not expect to see in the high projected growth area, why? Barak is going to buy 550 nails from one of these companies. Nail Company50 nails4. 15 plus VAT at 20%Hammer Company25 nails2. 95Special offerBuy 100 get 25 freeHe wants to buy the nails at the cheaper cost. Where should he buy the nails, from the Nail Company or the Hammer Company? Calculate the ph of a buffered solution prepared by dissolving 21.5 g benzoic acid and 37.7 g sodium benzoate Describe the transformations f(x)= square root 4x Directions: make your stand on the following issues write your answer inside thebox1. heavy traffic in metro manila2. face-to-face learning in this time of pandemic Order these events that occur during secondary succession correctly from 1-5. Even though keisha is intelligent and mature, she is easily entangled in jonathanhathaway's trap. explain how this occurs, and discuss whether you think keisha'smistakes are realistic.i need a answer asap When one knows the true values x1 and x2 and has approximations X1 and X2 at hand, one can see where errors may arise. By viewing error as something to be added to an approximation to attain a true value, it follows that the error ei is related to Xi and xi as xi 5 Xi 1 ei (a) Show that the error in a sum X1 1 X2 is (x1 1 x2) 2 (X1 1 X2) 5 e1 1 e2 (b) Show that the error in a difference X1 2 X2 is (x1 2 x2) 2 (X1 2 X2) 5 e1 2 e2 (c) Show that the error in a product X1X2 is x1x2 2 X1X2 < X1X2 a e1 X1 1 e2 X2 b (d) Show that in a quotient X1yX2 the error is x1 x2 2 X1 X2 < X1 X2 a e1 X1 2 e2 X2 b The drought is really effecting our corn this season. we're crossing our fingers for rain!